Can someone explain to me the jump between steps in the bottom two lines of this proof (not yet totally complete)? The Next CEO of Stack OverflowDifferential equation - Green's TheoremGreen's Theorem and limits on y for flux$oint_gamma frac-yx^2+y^2dx+fracxx^2+y^2dy$ over two different pathsHow does Green's theorem apply here?Calculate the integral using Green's TheoremProof of Green's Theorem for Special Regions and Bounds of Integrating $fracpartial Mpartial y$Understanding Green's Theorem ProofVerification of Green's Theorem in a Simple CaseWhy dosen't the author consider $oint_gamma_R f(z) = 2 pi i sum_j Indgamma(Pj) cdot Res_f(P_j)$?About the proof that $int_0^inftyfracdxx^2+6x+8 =frac12log2$ via residue formula

Why didn't Khan get resurrected in the Genesis Explosion?

Is there a difference between "Fahrstuhl" and "Aufzug"

Can you replace a racial trait cantrip when leveling up?

Why do remote companies require working in the US?

How do scammers retract money, while you can’t?

Return the Closest Prime Number

What flight has the highest ratio of time difference to flight time?

How does the Z80 determine which peripheral sent an interrupt?

How did people program for Consoles with multiple CPUs?

Why does the UK parliament need a vote on the political declaration?

Rotate a column

If/When UK leaves the EU, can a future goverment conduct a referendum to join the EU?

Is it possible to search for a directory/file combination?

Is it ever safe to open a suspicious html file (e.g. email attachment)?

Why has the US not been more assertive in confronting Russia in recent years?

What happens if you roll doubles 3 times then land on "Go to jail?"

Several mode to write the symbol of a vector

If the heap is zero-initialized for security, then why is the stack merely uninitialized?

How do I reset passwords on multiple websites easily?

Is "for causing autism in X" grammatical?

Would a galaxy be visible from outside, but nearby?

I believe this to be a fraud - hired, then asked to cash check and send cash as Bitcoin

Make solar eclipses exceedingly rare, but still have new moons

Grabbing quick drinks



Can someone explain to me the jump between steps in the bottom two lines of this proof (not yet totally complete)?



The Next CEO of Stack OverflowDifferential equation - Green's TheoremGreen's Theorem and limits on y for flux$oint_gamma frac-yx^2+y^2dx+fracxx^2+y^2dy$ over two different pathsHow does Green's theorem apply here?Calculate the integral using Green's TheoremProof of Green's Theorem for Special Regions and Bounds of Integrating $fracpartial Mpartial y$Understanding Green's Theorem ProofVerification of Green's Theorem in a Simple CaseWhy dosen't the author consider $oint_gamma_R f(z) = 2 pi i sum_j Indgamma(Pj) cdot Res_f(P_j)$?About the proof that $int_0^inftyfracdxx^2+6x+8 =frac12log2$ via residue formula










0












$begingroup$


Suppose we have a region $G$ that is bounded by the straight lines $x=a$, $x=b$, $y=c$ and by an arc $y = f(x)$ (which lies above $y=c$) where $a leq x leq b$.
If $f$, $P(x,y)$ and $Q(x,y)$ are all continuously differentiable in a region which contains the region $G$ and its boundary, denoted as $partial G$, then:



$$iint_G Bigg(fracpartial Qpartial x-fracpartial Ppartial yBigg) dA = oint_partial G Big(P cdot dx + Q cdot dyBig)$$



We can prove this lemma by showing the following integrals, which are taken by splitting up the above equality, are as such:
beginequation
iint_G Bigg(fracpartial Qpartial xBigg) cdot dA = oint_partial GQ cdot dy
endequation

beginequation
-iint_G Bigg(fracpartial Ppartial yBigg) cdot dA = oint_partial GP cdot dx
endequation

First, we shall prove the bottom equation. First we need to manipulate the double integral using the bounds given and then apply the Fundamental Theorem of Calculus. This will give us the following:
beginalign
-iint_G Bigg(fracpartial Ppartial yBigg) cdot dA &= - int_a^b int_c^f(x) fracpartial Ppartial y cdot dy cdot dx labeleqn:27 \
& = - int_a^b Big(P[x,f(x)] - P[x,c]Big) cdot dx labeleqn:28
endalign

Letting $oversetrightharpoonupC_3$ be the same as $y = f(x)$ and $oversetrightharpoonupC_1$ be the same as $y=c$ we can rewrite the line above as:
$$int_oversetrightharpoonupC_3P(x,y) cdot dx + int_oversetrightharpoonupC_1P(x,y) cdot dx$$



How does: $- int_a^b Big(P[x,f(x)] - P[x,c]Big) cdot dx = int_oversetrightharpoonupC_3P(x,y) cdot dx + int_oversetrightharpoonupC_1P(x,y) cdot dx$ ???



Apologies I couldn't get the anti clockwise arrow on the integrals for the contours.










share|cite|improve this question









$endgroup$
















    0












    $begingroup$


    Suppose we have a region $G$ that is bounded by the straight lines $x=a$, $x=b$, $y=c$ and by an arc $y = f(x)$ (which lies above $y=c$) where $a leq x leq b$.
    If $f$, $P(x,y)$ and $Q(x,y)$ are all continuously differentiable in a region which contains the region $G$ and its boundary, denoted as $partial G$, then:



    $$iint_G Bigg(fracpartial Qpartial x-fracpartial Ppartial yBigg) dA = oint_partial G Big(P cdot dx + Q cdot dyBig)$$



    We can prove this lemma by showing the following integrals, which are taken by splitting up the above equality, are as such:
    beginequation
    iint_G Bigg(fracpartial Qpartial xBigg) cdot dA = oint_partial GQ cdot dy
    endequation

    beginequation
    -iint_G Bigg(fracpartial Ppartial yBigg) cdot dA = oint_partial GP cdot dx
    endequation

    First, we shall prove the bottom equation. First we need to manipulate the double integral using the bounds given and then apply the Fundamental Theorem of Calculus. This will give us the following:
    beginalign
    -iint_G Bigg(fracpartial Ppartial yBigg) cdot dA &= - int_a^b int_c^f(x) fracpartial Ppartial y cdot dy cdot dx labeleqn:27 \
    & = - int_a^b Big(P[x,f(x)] - P[x,c]Big) cdot dx labeleqn:28
    endalign

    Letting $oversetrightharpoonupC_3$ be the same as $y = f(x)$ and $oversetrightharpoonupC_1$ be the same as $y=c$ we can rewrite the line above as:
    $$int_oversetrightharpoonupC_3P(x,y) cdot dx + int_oversetrightharpoonupC_1P(x,y) cdot dx$$



    How does: $- int_a^b Big(P[x,f(x)] - P[x,c]Big) cdot dx = int_oversetrightharpoonupC_3P(x,y) cdot dx + int_oversetrightharpoonupC_1P(x,y) cdot dx$ ???



    Apologies I couldn't get the anti clockwise arrow on the integrals for the contours.










    share|cite|improve this question









    $endgroup$














      0












      0








      0





      $begingroup$


      Suppose we have a region $G$ that is bounded by the straight lines $x=a$, $x=b$, $y=c$ and by an arc $y = f(x)$ (which lies above $y=c$) where $a leq x leq b$.
      If $f$, $P(x,y)$ and $Q(x,y)$ are all continuously differentiable in a region which contains the region $G$ and its boundary, denoted as $partial G$, then:



      $$iint_G Bigg(fracpartial Qpartial x-fracpartial Ppartial yBigg) dA = oint_partial G Big(P cdot dx + Q cdot dyBig)$$



      We can prove this lemma by showing the following integrals, which are taken by splitting up the above equality, are as such:
      beginequation
      iint_G Bigg(fracpartial Qpartial xBigg) cdot dA = oint_partial GQ cdot dy
      endequation

      beginequation
      -iint_G Bigg(fracpartial Ppartial yBigg) cdot dA = oint_partial GP cdot dx
      endequation

      First, we shall prove the bottom equation. First we need to manipulate the double integral using the bounds given and then apply the Fundamental Theorem of Calculus. This will give us the following:
      beginalign
      -iint_G Bigg(fracpartial Ppartial yBigg) cdot dA &= - int_a^b int_c^f(x) fracpartial Ppartial y cdot dy cdot dx labeleqn:27 \
      & = - int_a^b Big(P[x,f(x)] - P[x,c]Big) cdot dx labeleqn:28
      endalign

      Letting $oversetrightharpoonupC_3$ be the same as $y = f(x)$ and $oversetrightharpoonupC_1$ be the same as $y=c$ we can rewrite the line above as:
      $$int_oversetrightharpoonupC_3P(x,y) cdot dx + int_oversetrightharpoonupC_1P(x,y) cdot dx$$



      How does: $- int_a^b Big(P[x,f(x)] - P[x,c]Big) cdot dx = int_oversetrightharpoonupC_3P(x,y) cdot dx + int_oversetrightharpoonupC_1P(x,y) cdot dx$ ???



      Apologies I couldn't get the anti clockwise arrow on the integrals for the contours.










      share|cite|improve this question









      $endgroup$




      Suppose we have a region $G$ that is bounded by the straight lines $x=a$, $x=b$, $y=c$ and by an arc $y = f(x)$ (which lies above $y=c$) where $a leq x leq b$.
      If $f$, $P(x,y)$ and $Q(x,y)$ are all continuously differentiable in a region which contains the region $G$ and its boundary, denoted as $partial G$, then:



      $$iint_G Bigg(fracpartial Qpartial x-fracpartial Ppartial yBigg) dA = oint_partial G Big(P cdot dx + Q cdot dyBig)$$



      We can prove this lemma by showing the following integrals, which are taken by splitting up the above equality, are as such:
      beginequation
      iint_G Bigg(fracpartial Qpartial xBigg) cdot dA = oint_partial GQ cdot dy
      endequation

      beginequation
      -iint_G Bigg(fracpartial Ppartial yBigg) cdot dA = oint_partial GP cdot dx
      endequation

      First, we shall prove the bottom equation. First we need to manipulate the double integral using the bounds given and then apply the Fundamental Theorem of Calculus. This will give us the following:
      beginalign
      -iint_G Bigg(fracpartial Ppartial yBigg) cdot dA &= - int_a^b int_c^f(x) fracpartial Ppartial y cdot dy cdot dx labeleqn:27 \
      & = - int_a^b Big(P[x,f(x)] - P[x,c]Big) cdot dx labeleqn:28
      endalign

      Letting $oversetrightharpoonupC_3$ be the same as $y = f(x)$ and $oversetrightharpoonupC_1$ be the same as $y=c$ we can rewrite the line above as:
      $$int_oversetrightharpoonupC_3P(x,y) cdot dx + int_oversetrightharpoonupC_1P(x,y) cdot dx$$



      How does: $- int_a^b Big(P[x,f(x)] - P[x,c]Big) cdot dx = int_oversetrightharpoonupC_3P(x,y) cdot dx + int_oversetrightharpoonupC_1P(x,y) cdot dx$ ???



      Apologies I couldn't get the anti clockwise arrow on the integrals for the contours.







      proof-explanation greens-theorem






      share|cite|improve this question













      share|cite|improve this question











      share|cite|improve this question




      share|cite|improve this question










      asked Mar 18 at 22:13









      KeighleyiteKeighleyite

      789




      789




















          0






          active

          oldest

          votes












          Your Answer





          StackExchange.ifUsing("editor", function ()
          return StackExchange.using("mathjaxEditing", function ()
          StackExchange.MarkdownEditor.creationCallbacks.add(function (editor, postfix)
          StackExchange.mathjaxEditing.prepareWmdForMathJax(editor, postfix, [["$", "$"], ["\\(","\\)"]]);
          );
          );
          , "mathjax-editing");

          StackExchange.ready(function()
          var channelOptions =
          tags: "".split(" "),
          id: "69"
          ;
          initTagRenderer("".split(" "), "".split(" "), channelOptions);

          StackExchange.using("externalEditor", function()
          // Have to fire editor after snippets, if snippets enabled
          if (StackExchange.settings.snippets.snippetsEnabled)
          StackExchange.using("snippets", function()
          createEditor();
          );

          else
          createEditor();

          );

          function createEditor()
          StackExchange.prepareEditor(
          heartbeatType: 'answer',
          autoActivateHeartbeat: false,
          convertImagesToLinks: true,
          noModals: true,
          showLowRepImageUploadWarning: true,
          reputationToPostImages: 10,
          bindNavPrevention: true,
          postfix: "",
          imageUploader:
          brandingHtml: "Powered by u003ca class="icon-imgur-white" href="https://imgur.com/"u003eu003c/au003e",
          contentPolicyHtml: "User contributions licensed under u003ca href="https://creativecommons.org/licenses/by-sa/3.0/"u003ecc by-sa 3.0 with attribution requiredu003c/au003e u003ca href="https://stackoverflow.com/legal/content-policy"u003e(content policy)u003c/au003e",
          allowUrls: true
          ,
          noCode: true, onDemand: true,
          discardSelector: ".discard-answer"
          ,immediatelyShowMarkdownHelp:true
          );



          );













          draft saved

          draft discarded


















          StackExchange.ready(
          function ()
          StackExchange.openid.initPostLogin('.new-post-login', 'https%3a%2f%2fmath.stackexchange.com%2fquestions%2f3153386%2fcan-someone-explain-to-me-the-jump-between-steps-in-the-bottom-two-lines-of-this%23new-answer', 'question_page');

          );

          Post as a guest















          Required, but never shown

























          0






          active

          oldest

          votes








          0






          active

          oldest

          votes









          active

          oldest

          votes






          active

          oldest

          votes















          draft saved

          draft discarded
















































          Thanks for contributing an answer to Mathematics Stack Exchange!


          • Please be sure to answer the question. Provide details and share your research!

          But avoid


          • Asking for help, clarification, or responding to other answers.

          • Making statements based on opinion; back them up with references or personal experience.

          Use MathJax to format equations. MathJax reference.


          To learn more, see our tips on writing great answers.




          draft saved


          draft discarded














          StackExchange.ready(
          function ()
          StackExchange.openid.initPostLogin('.new-post-login', 'https%3a%2f%2fmath.stackexchange.com%2fquestions%2f3153386%2fcan-someone-explain-to-me-the-jump-between-steps-in-the-bottom-two-lines-of-this%23new-answer', 'question_page');

          );

          Post as a guest















          Required, but never shown





















































          Required, but never shown














          Required, but never shown












          Required, but never shown







          Required, but never shown

































          Required, but never shown














          Required, but never shown












          Required, but never shown







          Required, but never shown







          Popular posts from this blog

          Lowndes Grove History Architecture References Navigation menu32°48′6″N 79°57′58″W / 32.80167°N 79.96611°W / 32.80167; -79.9661132°48′6″N 79°57′58″W / 32.80167°N 79.96611°W / 32.80167; -79.9661178002500"National Register Information System"Historic houses of South Carolina"Lowndes Grove""+32° 48' 6.00", −79° 57' 58.00""Lowndes Grove, Charleston County (260 St. Margaret St., Charleston)""Lowndes Grove"The Charleston ExpositionIt Happened in South Carolina"Lowndes Grove (House), Saint Margaret Street & Sixth Avenue, Charleston, Charleston County, SC(Photographs)"Plantations of the Carolina Low Countrye

          random experiment with two different functions on unit interval Announcing the arrival of Valued Associate #679: Cesar Manara Planned maintenance scheduled April 23, 2019 at 00:00UTC (8:00pm US/Eastern)Random variable and probability space notionsRandom Walk with EdgesFinding functions where the increase over a random interval is Poisson distributedNumber of days until dayCan an observed event in fact be of zero probability?Unit random processmodels of coins and uniform distributionHow to get the number of successes given $n$ trials , probability $P$ and a random variable $X$Absorbing Markov chain in a computer. Is “almost every” turned into always convergence in computer executions?Stopped random walk is not uniformly integrable

          How should I support this large drywall patch? Planned maintenance scheduled April 23, 2019 at 00:00UTC (8:00pm US/Eastern) Announcing the arrival of Valued Associate #679: Cesar Manara Unicorn Meta Zoo #1: Why another podcast?How do I cover large gaps in drywall?How do I keep drywall around a patch from crumbling?Can I glue a second layer of drywall?How to patch long strip on drywall?Large drywall patch: how to avoid bulging seams?Drywall Mesh Patch vs. Bulge? To remove or not to remove?How to fix this drywall job?Prep drywall before backsplashWhat's the best way to fix this horrible drywall patch job?Drywall patching using 3M Patch Plus Primer